Simplify the complex expression…












0














I am asked to simplify the complex expression $$frac{1}{2}(|{e^{i{theta}}-1}^2|+|{e^{i{theta}}+1}|)$$



I have gotten to $$frac{1}{2}((2-2costheta)+(2+2costheta))$$
1. Do I expand to get $$frac{1}{2}(4)$$ OR
2. Do I factor out the 2 and get $$frac{1}{2}(2(1-costheta)+2(1+costheta))$$



Is this answer complete?
Additional question: How would the method change if the argument was negative? i.e. if it was $$frac{1}{2}(|{e^{-i{theta}}-1}^2|+|{e^{-i{theta}}+1}|)$$










share|cite|improve this question
























  • What is $operatorname{mod}$ in this context?
    – José Carlos Santos
    Nov 21 '18 at 10:34










  • It is meant to be Modulus, like the absolute value... I wasnt sure how else to do it. It should be around both terms.
    – Kayla Martin
    Nov 21 '18 at 10:48












  • Do you mean $lvert e^{itheta}-1rvert$?
    – José Carlos Santos
    Nov 21 '18 at 10:51










  • Yes, exactly. Same for the +1 term.
    – Kayla Martin
    Nov 21 '18 at 10:52










  • Then why don't you just type |e^{itheta}-1|?
    – José Carlos Santos
    Nov 21 '18 at 10:54
















0














I am asked to simplify the complex expression $$frac{1}{2}(|{e^{i{theta}}-1}^2|+|{e^{i{theta}}+1}|)$$



I have gotten to $$frac{1}{2}((2-2costheta)+(2+2costheta))$$
1. Do I expand to get $$frac{1}{2}(4)$$ OR
2. Do I factor out the 2 and get $$frac{1}{2}(2(1-costheta)+2(1+costheta))$$



Is this answer complete?
Additional question: How would the method change if the argument was negative? i.e. if it was $$frac{1}{2}(|{e^{-i{theta}}-1}^2|+|{e^{-i{theta}}+1}|)$$










share|cite|improve this question
























  • What is $operatorname{mod}$ in this context?
    – José Carlos Santos
    Nov 21 '18 at 10:34










  • It is meant to be Modulus, like the absolute value... I wasnt sure how else to do it. It should be around both terms.
    – Kayla Martin
    Nov 21 '18 at 10:48












  • Do you mean $lvert e^{itheta}-1rvert$?
    – José Carlos Santos
    Nov 21 '18 at 10:51










  • Yes, exactly. Same for the +1 term.
    – Kayla Martin
    Nov 21 '18 at 10:52










  • Then why don't you just type |e^{itheta}-1|?
    – José Carlos Santos
    Nov 21 '18 at 10:54














0












0








0







I am asked to simplify the complex expression $$frac{1}{2}(|{e^{i{theta}}-1}^2|+|{e^{i{theta}}+1}|)$$



I have gotten to $$frac{1}{2}((2-2costheta)+(2+2costheta))$$
1. Do I expand to get $$frac{1}{2}(4)$$ OR
2. Do I factor out the 2 and get $$frac{1}{2}(2(1-costheta)+2(1+costheta))$$



Is this answer complete?
Additional question: How would the method change if the argument was negative? i.e. if it was $$frac{1}{2}(|{e^{-i{theta}}-1}^2|+|{e^{-i{theta}}+1}|)$$










share|cite|improve this question















I am asked to simplify the complex expression $$frac{1}{2}(|{e^{i{theta}}-1}^2|+|{e^{i{theta}}+1}|)$$



I have gotten to $$frac{1}{2}((2-2costheta)+(2+2costheta))$$
1. Do I expand to get $$frac{1}{2}(4)$$ OR
2. Do I factor out the 2 and get $$frac{1}{2}(2(1-costheta)+2(1+costheta))$$



Is this answer complete?
Additional question: How would the method change if the argument was negative? i.e. if it was $$frac{1}{2}(|{e^{-i{theta}}-1}^2|+|{e^{-i{theta}}+1}|)$$







calculus complex-analysis analysis complex-numbers polar-coordinates






share|cite|improve this question















share|cite|improve this question













share|cite|improve this question




share|cite|improve this question








edited Nov 22 '18 at 4:44









naveen dankal

4,52821348




4,52821348










asked Nov 21 '18 at 10:33









Kayla Martin

636




636












  • What is $operatorname{mod}$ in this context?
    – José Carlos Santos
    Nov 21 '18 at 10:34










  • It is meant to be Modulus, like the absolute value... I wasnt sure how else to do it. It should be around both terms.
    – Kayla Martin
    Nov 21 '18 at 10:48












  • Do you mean $lvert e^{itheta}-1rvert$?
    – José Carlos Santos
    Nov 21 '18 at 10:51










  • Yes, exactly. Same for the +1 term.
    – Kayla Martin
    Nov 21 '18 at 10:52










  • Then why don't you just type |e^{itheta}-1|?
    – José Carlos Santos
    Nov 21 '18 at 10:54


















  • What is $operatorname{mod}$ in this context?
    – José Carlos Santos
    Nov 21 '18 at 10:34










  • It is meant to be Modulus, like the absolute value... I wasnt sure how else to do it. It should be around both terms.
    – Kayla Martin
    Nov 21 '18 at 10:48












  • Do you mean $lvert e^{itheta}-1rvert$?
    – José Carlos Santos
    Nov 21 '18 at 10:51










  • Yes, exactly. Same for the +1 term.
    – Kayla Martin
    Nov 21 '18 at 10:52










  • Then why don't you just type |e^{itheta}-1|?
    – José Carlos Santos
    Nov 21 '18 at 10:54
















What is $operatorname{mod}$ in this context?
– José Carlos Santos
Nov 21 '18 at 10:34




What is $operatorname{mod}$ in this context?
– José Carlos Santos
Nov 21 '18 at 10:34












It is meant to be Modulus, like the absolute value... I wasnt sure how else to do it. It should be around both terms.
– Kayla Martin
Nov 21 '18 at 10:48






It is meant to be Modulus, like the absolute value... I wasnt sure how else to do it. It should be around both terms.
– Kayla Martin
Nov 21 '18 at 10:48














Do you mean $lvert e^{itheta}-1rvert$?
– José Carlos Santos
Nov 21 '18 at 10:51




Do you mean $lvert e^{itheta}-1rvert$?
– José Carlos Santos
Nov 21 '18 at 10:51












Yes, exactly. Same for the +1 term.
– Kayla Martin
Nov 21 '18 at 10:52




Yes, exactly. Same for the +1 term.
– Kayla Martin
Nov 21 '18 at 10:52












Then why don't you just type |e^{itheta}-1|?
– José Carlos Santos
Nov 21 '18 at 10:54




Then why don't you just type |e^{itheta}-1|?
– José Carlos Santos
Nov 21 '18 at 10:54










1 Answer
1






active

oldest

votes


















0














The complet answer is $2$. That follows from both of your approaches. That is$$(forallthetainmathbb R):lvert e^{itheta}-1rvert^2+lvert e^{-itheta}-1rvert^2=2.$$Note that this is for all real numbers $theta$. Therefore, it makes no sense to ask what happens if we deal with $-theta$ instead of $theta$.






share|cite|improve this answer























  • I have a different question that is exactly the same as this but the argument is negative (like I added toward the bottom of the question) so that is why I was asking. Are you saying that the answer will be the same for both?
    – Kayla Martin
    Nov 21 '18 at 12:04












  • If you prove something for an arbitrary $thetainmathbb R$, you shall have proved it both for positive and for negative real numbers.
    – José Carlos Santos
    Nov 21 '18 at 12:10










  • Is this because of the modulus in this question? I don't quite get what you mean sorry. I am doing past exam papers so I have no answers to refer to either.
    – Kayla Martin
    Nov 21 '18 at 12:14










  • I cannot help you here, since I don't understand where your doubt is.
    – José Carlos Santos
    Nov 21 '18 at 12:35










  • Is it a different answer for a negative argument or is it the same if everything else in the question is the same?
    – Kayla Martin
    Nov 21 '18 at 12:39











Your Answer





StackExchange.ifUsing("editor", function () {
return StackExchange.using("mathjaxEditing", function () {
StackExchange.MarkdownEditor.creationCallbacks.add(function (editor, postfix) {
StackExchange.mathjaxEditing.prepareWmdForMathJax(editor, postfix, [["$", "$"], ["\\(","\\)"]]);
});
});
}, "mathjax-editing");

StackExchange.ready(function() {
var channelOptions = {
tags: "".split(" "),
id: "69"
};
initTagRenderer("".split(" "), "".split(" "), channelOptions);

StackExchange.using("externalEditor", function() {
// Have to fire editor after snippets, if snippets enabled
if (StackExchange.settings.snippets.snippetsEnabled) {
StackExchange.using("snippets", function() {
createEditor();
});
}
else {
createEditor();
}
});

function createEditor() {
StackExchange.prepareEditor({
heartbeatType: 'answer',
autoActivateHeartbeat: false,
convertImagesToLinks: true,
noModals: true,
showLowRepImageUploadWarning: true,
reputationToPostImages: 10,
bindNavPrevention: true,
postfix: "",
imageUploader: {
brandingHtml: "Powered by u003ca class="icon-imgur-white" href="https://imgur.com/"u003eu003c/au003e",
contentPolicyHtml: "User contributions licensed under u003ca href="https://creativecommons.org/licenses/by-sa/3.0/"u003ecc by-sa 3.0 with attribution requiredu003c/au003e u003ca href="https://stackoverflow.com/legal/content-policy"u003e(content policy)u003c/au003e",
allowUrls: true
},
noCode: true, onDemand: true,
discardSelector: ".discard-answer"
,immediatelyShowMarkdownHelp:true
});


}
});














draft saved

draft discarded


















StackExchange.ready(
function () {
StackExchange.openid.initPostLogin('.new-post-login', 'https%3a%2f%2fmath.stackexchange.com%2fquestions%2f3007539%2fsimplify-the-complex-expression%23new-answer', 'question_page');
}
);

Post as a guest















Required, but never shown

























1 Answer
1






active

oldest

votes








1 Answer
1






active

oldest

votes









active

oldest

votes






active

oldest

votes









0














The complet answer is $2$. That follows from both of your approaches. That is$$(forallthetainmathbb R):lvert e^{itheta}-1rvert^2+lvert e^{-itheta}-1rvert^2=2.$$Note that this is for all real numbers $theta$. Therefore, it makes no sense to ask what happens if we deal with $-theta$ instead of $theta$.






share|cite|improve this answer























  • I have a different question that is exactly the same as this but the argument is negative (like I added toward the bottom of the question) so that is why I was asking. Are you saying that the answer will be the same for both?
    – Kayla Martin
    Nov 21 '18 at 12:04












  • If you prove something for an arbitrary $thetainmathbb R$, you shall have proved it both for positive and for negative real numbers.
    – José Carlos Santos
    Nov 21 '18 at 12:10










  • Is this because of the modulus in this question? I don't quite get what you mean sorry. I am doing past exam papers so I have no answers to refer to either.
    – Kayla Martin
    Nov 21 '18 at 12:14










  • I cannot help you here, since I don't understand where your doubt is.
    – José Carlos Santos
    Nov 21 '18 at 12:35










  • Is it a different answer for a negative argument or is it the same if everything else in the question is the same?
    – Kayla Martin
    Nov 21 '18 at 12:39
















0














The complet answer is $2$. That follows from both of your approaches. That is$$(forallthetainmathbb R):lvert e^{itheta}-1rvert^2+lvert e^{-itheta}-1rvert^2=2.$$Note that this is for all real numbers $theta$. Therefore, it makes no sense to ask what happens if we deal with $-theta$ instead of $theta$.






share|cite|improve this answer























  • I have a different question that is exactly the same as this but the argument is negative (like I added toward the bottom of the question) so that is why I was asking. Are you saying that the answer will be the same for both?
    – Kayla Martin
    Nov 21 '18 at 12:04












  • If you prove something for an arbitrary $thetainmathbb R$, you shall have proved it both for positive and for negative real numbers.
    – José Carlos Santos
    Nov 21 '18 at 12:10










  • Is this because of the modulus in this question? I don't quite get what you mean sorry. I am doing past exam papers so I have no answers to refer to either.
    – Kayla Martin
    Nov 21 '18 at 12:14










  • I cannot help you here, since I don't understand where your doubt is.
    – José Carlos Santos
    Nov 21 '18 at 12:35










  • Is it a different answer for a negative argument or is it the same if everything else in the question is the same?
    – Kayla Martin
    Nov 21 '18 at 12:39














0












0








0






The complet answer is $2$. That follows from both of your approaches. That is$$(forallthetainmathbb R):lvert e^{itheta}-1rvert^2+lvert e^{-itheta}-1rvert^2=2.$$Note that this is for all real numbers $theta$. Therefore, it makes no sense to ask what happens if we deal with $-theta$ instead of $theta$.






share|cite|improve this answer














The complet answer is $2$. That follows from both of your approaches. That is$$(forallthetainmathbb R):lvert e^{itheta}-1rvert^2+lvert e^{-itheta}-1rvert^2=2.$$Note that this is for all real numbers $theta$. Therefore, it makes no sense to ask what happens if we deal with $-theta$ instead of $theta$.







share|cite|improve this answer














share|cite|improve this answer



share|cite|improve this answer








edited Nov 21 '18 at 12:35

























answered Nov 21 '18 at 11:41









José Carlos Santos

151k22123224




151k22123224












  • I have a different question that is exactly the same as this but the argument is negative (like I added toward the bottom of the question) so that is why I was asking. Are you saying that the answer will be the same for both?
    – Kayla Martin
    Nov 21 '18 at 12:04












  • If you prove something for an arbitrary $thetainmathbb R$, you shall have proved it both for positive and for negative real numbers.
    – José Carlos Santos
    Nov 21 '18 at 12:10










  • Is this because of the modulus in this question? I don't quite get what you mean sorry. I am doing past exam papers so I have no answers to refer to either.
    – Kayla Martin
    Nov 21 '18 at 12:14










  • I cannot help you here, since I don't understand where your doubt is.
    – José Carlos Santos
    Nov 21 '18 at 12:35










  • Is it a different answer for a negative argument or is it the same if everything else in the question is the same?
    – Kayla Martin
    Nov 21 '18 at 12:39


















  • I have a different question that is exactly the same as this but the argument is negative (like I added toward the bottom of the question) so that is why I was asking. Are you saying that the answer will be the same for both?
    – Kayla Martin
    Nov 21 '18 at 12:04












  • If you prove something for an arbitrary $thetainmathbb R$, you shall have proved it both for positive and for negative real numbers.
    – José Carlos Santos
    Nov 21 '18 at 12:10










  • Is this because of the modulus in this question? I don't quite get what you mean sorry. I am doing past exam papers so I have no answers to refer to either.
    – Kayla Martin
    Nov 21 '18 at 12:14










  • I cannot help you here, since I don't understand where your doubt is.
    – José Carlos Santos
    Nov 21 '18 at 12:35










  • Is it a different answer for a negative argument or is it the same if everything else in the question is the same?
    – Kayla Martin
    Nov 21 '18 at 12:39
















I have a different question that is exactly the same as this but the argument is negative (like I added toward the bottom of the question) so that is why I was asking. Are you saying that the answer will be the same for both?
– Kayla Martin
Nov 21 '18 at 12:04






I have a different question that is exactly the same as this but the argument is negative (like I added toward the bottom of the question) so that is why I was asking. Are you saying that the answer will be the same for both?
– Kayla Martin
Nov 21 '18 at 12:04














If you prove something for an arbitrary $thetainmathbb R$, you shall have proved it both for positive and for negative real numbers.
– José Carlos Santos
Nov 21 '18 at 12:10




If you prove something for an arbitrary $thetainmathbb R$, you shall have proved it both for positive and for negative real numbers.
– José Carlos Santos
Nov 21 '18 at 12:10












Is this because of the modulus in this question? I don't quite get what you mean sorry. I am doing past exam papers so I have no answers to refer to either.
– Kayla Martin
Nov 21 '18 at 12:14




Is this because of the modulus in this question? I don't quite get what you mean sorry. I am doing past exam papers so I have no answers to refer to either.
– Kayla Martin
Nov 21 '18 at 12:14












I cannot help you here, since I don't understand where your doubt is.
– José Carlos Santos
Nov 21 '18 at 12:35




I cannot help you here, since I don't understand where your doubt is.
– José Carlos Santos
Nov 21 '18 at 12:35












Is it a different answer for a negative argument or is it the same if everything else in the question is the same?
– Kayla Martin
Nov 21 '18 at 12:39




Is it a different answer for a negative argument or is it the same if everything else in the question is the same?
– Kayla Martin
Nov 21 '18 at 12:39


















draft saved

draft discarded




















































Thanks for contributing an answer to Mathematics Stack Exchange!


  • Please be sure to answer the question. Provide details and share your research!

But avoid



  • Asking for help, clarification, or responding to other answers.

  • Making statements based on opinion; back them up with references or personal experience.


Use MathJax to format equations. MathJax reference.


To learn more, see our tips on writing great answers.





Some of your past answers have not been well-received, and you're in danger of being blocked from answering.


Please pay close attention to the following guidance:


  • Please be sure to answer the question. Provide details and share your research!

But avoid



  • Asking for help, clarification, or responding to other answers.

  • Making statements based on opinion; back them up with references or personal experience.


To learn more, see our tips on writing great answers.




draft saved


draft discarded














StackExchange.ready(
function () {
StackExchange.openid.initPostLogin('.new-post-login', 'https%3a%2f%2fmath.stackexchange.com%2fquestions%2f3007539%2fsimplify-the-complex-expression%23new-answer', 'question_page');
}
);

Post as a guest















Required, but never shown





















































Required, but never shown














Required, but never shown












Required, but never shown







Required, but never shown

































Required, but never shown














Required, but never shown












Required, but never shown







Required, but never shown







Popular posts from this blog

Can a sorcerer learn a 5th-level spell early by creating spell slots using the Font of Magic feature?

Does disintegrating a polymorphed enemy still kill it after the 2018 errata?

A Topological Invariant for $pi_3(U(n))$